3
$\begingroup$

Let $f:\mathbb{R}^+\to \mathbb{R}^+$ be given by

$f(t) = 1$ if $0\leq t\leq 1$

$f(t) = 1/t^2$ if $t>1$.

Question:

What is the sum $\sum_{n\; \text{squarefree}} f(n/x)$, for $x$ large?

Just by the simplest inclusion-exclusion, one can get an estimate of the form $x/\zeta(2) + C \sqrt{x}$, where $C$ is a largish constant, in part by using highly optimized estimates for $\sum_{n\leq x: n\; \text{squarefree}} 1$ in the literature (e.g. Cohen and Dress, MR0952866). However, I'd imagine the smoothing inherent in the problem allows one to do better than that. How much better? (Experiments suggest a very small error term.)

$\endgroup$
10
  • 4
    $\begingroup$ There must be some typo since the sum in the question is $+ \infty$. $\endgroup$
    – user25235
    Jan 28, 2013 at 15:24
  • 1
    $\begingroup$ Since the generating function for square-free numbers is $\zeta(s)/\zeta(2s)$, I believe you can get an error of $o(\sqrt{x})$ by using the classical zero-free region for the zeta-function (even without smoothing the sum). To get an error like $O(x^{1/2-\delta})$ for some $\delta>0$ seems more or less equivalent to a quasi-Riemann hypothesis. $\endgroup$ Jan 28, 2013 at 15:25
  • $\begingroup$ I'm with km. Indeed, for any set of integers n of positive density, I see the desired sum over that set as infinite. Gerhard "Ask Me About Unbounded Confusion" Paseman, 2013.01.28 $\endgroup$ Jan 28, 2013 at 16:48
  • 2
    $\begingroup$ FWIW, there's an update to the Cohen-Dress paper (by Cohen, Dress, and El Marraki) available through ams.org/mathscinet-getitem?mr=2357309 $\endgroup$ Jan 28, 2013 at 19:34
  • 1
    $\begingroup$ When I do the inclusion-exclusion, I get a main term of $2x/\zeta(2)$ rather than $x/\zeta(2)$ - each of the ranges $n\le x$ and $n>x$ seems to contribute an $x/\zeta(2)$. Do you think that's right? $\endgroup$ Jan 29, 2013 at 2:02

1 Answer 1

2
$\begingroup$

There are two approaches I can think of.

(a) Analytic. We need only information about $\zeta(s)$, not about other $L$-functions. Hence we can use the fact that RH has been verified up to a very large height. This should imply a result of the form

$\sum_{n\; \text{squarefree}} f(n/x) = (1+\epsilon) 2 \zeta(2) + O^*(C_{\epsilon'} x^{1/4+\epsilon'})$,

where $O^*(K)$ means " a quantity of absolute value at most $K$", $\epsilon$ is a very tiny constant, $\epsilon'>0$ is arbitrarily small and $C_{\epsilon'}$ depends only on $\epsilon'$.

Difficulty: giving an explicit, and preferably small, value for $C_{\epsilon'}$ does not seem very easy. We cannot shift the line of integration all the way to $\Re(s)=1/2$ without having a bound on the residues of $1/\zeta(s)$; if we shift the line of integration only to $\Re(s)=1/2+\epsilon'$, we still need upper bounds for $1/|\zeta(s)|$ (i.e., lower bounds for $\zeta(s)$). There is some non-explicit work in this direction, but I do not know of anything explicit with reasonable constants. (I would be very glad to be surprised.)

(b) "Elementary". A fully elementary bound would presumably follow Cohen and Dress, MR0952866, and give bounds of about the same quality (error term = $O(x^{1/2})$, where the implied constant is not large but also not very small). A more mixed approach would proceed as in the paper by Cohen, Dress and El Marraki cited by Barry Cipra above. Half of the work (sieving out factors m^2 with $m\leq c\cdot \sqrt{x}$) would indeed be improved by the smoothing. The other half would rely on estimates on $M(N) = \sum_{n\leq N} \mu(n)$. Here estimates of the form $|M(N)|\leq c N$, $c$ a small constant (for $N$ larger than a constant) are known, and lead to estimates of the form

$\sum_{n\; \text{squarefree}} f(n/x) = 2 \zeta(2) + O^*(c x^{1/2})$,

with $c$ rather small (but not "very tiny"; we are speaking about $10^{-3}$ rather than $10^{-20}$ , say) provided that $x$ is larger than a constant. Getting useful error-term estimates better than $O^*(c x^{1/2})$ is hard, however, in that it involves estimating $M(x)$: this seems hard for the same reason given above, namely, estimates for the residues of $1/\zeta(s)$ or upper bounds for $1/|\zeta(s)|$ become necessary.

Question: is there a third way? Or is there a way to give good upper bounds for $1/|\zeta(s)|$ for $|\Im(s)|\leq H_0$, assuming that the Riemann hypothesis holds for $|\Im(s)|\leq H_0$ (or a little further)?

$\endgroup$

Your Answer

By clicking “Post Your Answer”, you agree to our terms of service and acknowledge you have read our privacy policy.

Not the answer you're looking for? Browse other questions tagged or ask your own question.